How Do You Solve Defective Eigenvalues in Differential Equations?

  • Thread starter Thread starter µ³
  • Start date Start date
  • Tags Tags
    Eigenvalues
Click For Summary
To solve for the third eigenvector v3 in a differential equation with a defective eigenvalue, start by ensuring that v3 satisfies the equation (\mathbf{A}-\lambda \mathbf{I})^2 \mathbf{v_3} = \mathbf{0}. After finding v3, the general solution can be structured as a linear combination of the eigenvectors v1, v2, and v3, potentially including terms multiplied by t for the defective eigenvalue. A suggested form for the solution is (C_1 v_1 + C_2 t v_1 + C_3 v_2 + C_4 t v_2 + C_5 v_3 + C_6 t v_3)e^{\lambda t}. Testing this general solution in the original equation can help determine the relationships between the coefficients.
µ³
Messages
68
Reaction score
0
if you have a differential equation of the form
x' = Ax
where A is the coefficient matrix, and you get a triple eigenvalue with a defect of 1. (meaning you get v1 and v2 as the associated eigenvector). How do you get v3 and how do you set up the solutions?
I tried finding v3 such that (\mathbf{A}-\lambda \mathbf{I})^2 \mathbf{v_3} = \mathbf{0} but not exactly sure what to do after that. I got another v_2 (called it v_2') by using (\mathbf{A}-\lambda \mathbf{I}) \mathbf{v_3} = \mathbf{v_2'} but again not sure how to set up solution.
my answer was
\mathbf{x_1(t)} = \mathbf{v_1} e^{\lambda t}
\mathbf{x_2(t)} = \mathbf{v_2'} e^{\lambda t}a
\mathbf{x_3(t)} = (\mathbf{v_2'} + \mathbf{v_3}t)e^{\lambda t}
but it is not what is in the back of the book. I also tried using the original v_2:
\mathbf{x_1(t)} = \mathbf{v_1} e^{\lambda t}
\mathbf{x_2(t)} = \mathbf{v_2} e^{\lambda t}a
\mathbf{x_3(t)} = (\mathbf{v_2} + \mathbf{v_3}t)e^{\lambda t}
Can anyone help?
Thanks in advance.
 
Physics news on Phys.org
Well, it's awfully brutish, but if you think that the general solution can be written in terms of your v_1, v_2, and v_3, possibly with an extra factor of t on some terms, then why not just make the most general candidate possible:

(C_1 v_1 + C_2 t v_1 + C_3 v_2 + C_4 t v_2 + C_5 v_3 + C_6 t v_3)e^{\lambda t}

and plug it into the original equation, to see what relations must hold between the various coefficients?
 
Maybe it would help to examine the solutions to a simpler version of the same sort of problem. For example:

A = \left(\begin{array}{ccc}2&0&0\\0&2&1\\0&0&2\end{array}\right)

The difficulty is the same as for this problem:

A = \left(\begin{array}{cc}2&1\\0&2\end{array}\right)


Carl
 
Question: A clock's minute hand has length 4 and its hour hand has length 3. What is the distance between the tips at the moment when it is increasing most rapidly?(Putnam Exam Question) Answer: Making assumption that both the hands moves at constant angular velocities, the answer is ## \sqrt{7} .## But don't you think this assumption is somewhat doubtful and wrong?

Similar threads

  • · Replies 5 ·
Replies
5
Views
2K
  • · Replies 2 ·
Replies
2
Views
1K
  • · Replies 6 ·
Replies
6
Views
1K
  • · Replies 4 ·
Replies
4
Views
2K
Replies
8
Views
3K
  • · Replies 2 ·
Replies
2
Views
4K
Replies
6
Views
2K
  • · Replies 2 ·
Replies
2
Views
2K
  • · Replies 6 ·
Replies
6
Views
3K
  • · Replies 2 ·
Replies
2
Views
2K